Your-Doctor
Multiple Choice Questions (MCQ)



Free Palestine
Quiz Categories Click to expand

Category: Prometric--->Neurology
Page: 35

Question 171# Print Question

A 24-year-old woman with Charcot-Marie-Tooth disease (type 1) asks how likely it is that any future children will have the disease.

What is the most accurate answer?

A. Three times as likely as background population
B. 25%
C. Between 5 - 10%
D. Same as background population
E. 50%


Question 172# Print Question

A 44-year-old woman presents with a three month history of worsening involuntary movements of the head. These are worse when she is stressed and improved by alcohol. They are not present when she is sleep. There are no other neurological symptoms of note and neurological examination is unremarkable other than spontaneous movements of the head which are worse when she looks to either side. Her father had a similar complaint but never sought medical attention.

What is the most likely diagnosis?

A. Parkinson's disease
B. Cerebellar tremor
C. Huntington's disease
D. Multiple sclerosis
E. Essential tremor


Question 173# Print Question

A 40-year-old woman who is known to be HIV positive is admitted to the Emergency Department following a seizure. Her partner reports that she has been having headaches, night sweats and a poor appetite for the past four weeks. Blood tests and a CT head are arranged:

  • CD4 89 u/l
  • CT head Single ring-enhancing lesion in the right parietal lobe

What is the most likely diagnosis?

A. Primary CNS lymphoma
B. Tuberculosis
C. Progressive multifocal leukoencephalopathy
D. Cryptococcus
E. Toxoplasmosis


Question 174# Print Question

Which one of the following statements regarding the use of 5-HT1 agonists in the treatment of migraine is incorrect?

A. May be given subcutaneously
B. Are second line therapy in the management of acute migraine
C. Should be taken as soon as possible after the onset of an aura
D. Should be avoided in patients with ischaemic heart disease
E. Adverse effects include tingling and chest tightness


Question 175# Print Question

Which one of the following statements regarding epilepsy in pregnant women is correct?

A. All pregnant women on antiepileptic medication should take 400mcg a day of folic acid
B. Antiepileptic drug levels should be monitored throughout pregnancy
C. The dose of lamotrigine usually needs to be decreased during pregnancy
D. Pregnant women taking phenytoin should be given vitamin K in the last month of pregnancy
E. Sodium valproate is most strongly associated with cleft palate




Category: Prometric--->Neurology
Page: 35 of 41